Jump to content

Minhnguyenthe333's Content

There have been 788 items by Minhnguyenthe333 (Search limited from 24-05-2020)



Sort by                Order  

#647952 $a=b$

Posted by Minhnguyenthe333 on 04-08-2016 - 18:02 in Số học

(AoPS) Cho $a,b,m,n \in \mathbb{Z^+}$ và $n>1$ thỏa mãn $a^n+b^n=2^m$. Chứng minh rằng $a=b$




#647948 Tìm số $n\in \mathbb{N}^{*}$ nhỏ nhất...

Posted by Minhnguyenthe333 on 04-08-2016 - 17:42 in Số học

Tìm số $n\in \mathbb{N}^{*}$ nhỏ nhất sao cho $\frac{9^{n}-1^{n}}{4}$ chia hết cho $2^{2014}$

$2^{2014}\mid \frac{9^n-1^n}{4}\iff 2^{2016}\mid 9^n-1^n$

Áp dụng bổ đề LTE, ta có: $v_2(9^n-1^n)=v_2(8)+v_2(n)=3+v_2(n)$

$2^{2016}\mid 9^n-1^n\iff v_2(9^n-1^n)\geqslant v_2(2^{2016})=2016$

$\iff 3+v_2(n)\geqslant 2016\iff n\geqslant 2^{2013}$

Vậy $n_{min}= 2^{2013}$




#647600 Trại hè Hùng Vương 2016 Toán 10

Posted by Minhnguyenthe333 on 02-08-2016 - 09:36 in Thi HSG cấp Tỉnh, Thành phố. Olympic 30-4. Đề thi và kiểm tra đội tuyển các cấp.

Bài 5: Tìm các số $p,n $ thoả $p$ nguyên tố, $n$ là số nguyên dương sao cho

$p^3 -2p^2 + p +1= 3^n $

Giải theo kiểu này:

$\iff (p+1)(p^2-3p+4)=3(3^{n-1}+1)$

Đặt $d=\gcd(p+1,p^2-3p+4)\Longrightarrow d=\gcd(p+1,-5p+3)=\gcd(p+1,8)\iff d\mid 8$

Dễ thấy $\gcd(3,3^{n-1}+1)=1$ nên ta có các trường hợp sau:

$d=1\Longrightarrow \left\{\begin{matrix} p^2-3p+4=3^{n-1}+1\\ p+1=3 \end{matrix}\right.\iff (n,p)=(1,2)$

 

$d=2\Longrightarrow \left\{\begin{matrix} \frac{p^2-3p+4}{2}=\frac{3^{n-1}+1}{4}\\ \frac{p+1}{2}=3 \end{matrix}\right.\iff (n,p)=(4,5)$

 

$d=4\Longrightarrow \left\{\begin{matrix} \frac{p^2-3p+4}{4}=\frac{3^{n-1}+1}{16}\\ \frac{p+1}{4}=3 \end{matrix}\right.\iff PT$ vô nghiệm

 

$d=8\Longrightarrow \left\{\begin{matrix} \frac{p^2-3p+4}{8}=\frac{3^{n-1}+1}{64}\\ \frac{p+1}{8}=3 \end{matrix}\right.\iff PT$ vô nghiệm

 

Vậy $(n,p)=(1,2);(4,5)$




#647440 Trại hè Hùng Vương 2016 Toán 10

Posted by Minhnguyenthe333 on 31-07-2016 - 23:56 in Thi HSG cấp Tỉnh, Thành phố. Olympic 30-4. Đề thi và kiểm tra đội tuyển các cấp.

Bài 3:
Đổi biến $\left ( \frac{a}{a+b},\frac{b}{b+c},\frac{c}{c+a} \right )\rightarrow (1-x,1-y,1-z)$
Dễ thấy $xyz=(1-x)(1-y)(1-z)\iff 2xyz=1+\sum x-\sum xy$
BĐT$\iff \sum (1-x)^2+4xyz\geqslant 3-(x+y+z)+\frac{1}{4}$
$\iff x^2+y^2+z^2+4xyz\geqslant x+y+z-\frac{1}{4}$
Thay $2xyz=1+\sum x-\sum xy$
$\iff x^2+y^2+z^2+2+2(x+y+z)-2(xy+yz+zx)\geqslant x+y+z-\frac{1}{4}$
$\iff (x+y+z)^2-3(x+y+z)+\frac{9}{4}\geqslant 0\iff [2(x+y+z)-3]^2\geqslant 0$ (luôn đúng)
Dấu "=" xảy ra khi $\sum \frac{a}{a+b}=\frac{3}{2}\iff ....$



#647393 $\left | \sum \frac{a-b}{a+b} \r...

Posted by Minhnguyenthe333 on 31-07-2016 - 20:53 in Bất đẳng thức và cực trị

Tìm hằng số $k$ lớn nhất để bất đẳng thức sau đúng với mọi $a,b,c$ là độ dài 3 cạnh của một tam giác: 

                                          $\left | \frac{a-b}{a+b}+  \frac{b-c}{b+c}+ \frac{c-a}{c+a}\right |<\frac{1}{k}$




#647308 $x^2+y^2+x+y+1=xyz$

Posted by Minhnguyenthe333 on 31-07-2016 - 12:01 in Số học

Tìm tất cả bộ ba số tự nhiên $(x,y,z)$ thỏa mãn:  $x^2+y^2+x+y+1=xyz$




#647300 Chứng minh: $m^{2}-n^{2}+1$ là số chính phương.

Posted by Minhnguyenthe333 on 31-07-2016 - 11:04 in Số học

Cho $m,n$ là hai số lẻ với $m>n>1$ thoả mãn: $m^{2}-n^{2}+1|n^{2}-1$. Chứng minh: $m^{2}-n^{2}+1$ là số chính phương.

$m^2-n^2+1\mid n^2-1=-(m^2-n^2+1)+m^2 \iff m^2-n^2+1\mid m^2$

$\Longrightarrow $ Tồn tại số $k$ nguyên dương sao cho: $m^2=k(m^2-n^2+1)$ $(1)$

Do đó ta chỉ cần chứng minh $k$ là số chính phương

Đặt $x=\frac{m+n}{2}$ và $y=\frac{m-n}{2}$

$(1)\iff \left ( \frac{m+n}{2}+\frac{m-n}{2} \right )^2=k\left ( 4.\frac{(m+n)}{2}\frac{(m-n)}{2}+1 \right )$

$\iff (x+y)^2=k(4xy+1)\iff x^2+x(2y-4ky)+y^2-k=0$ $(2)$

Cố định tập nghiệm, giả sử $x\geqslant y$ và $x+y$ nhỏ nhất

Theo Vieta, ngoài nghiệm $x$ thì $(2)$ còn nghiệm $t$ nguyên thỏa mãn: $\left\{\begin{matrix} t+x=4ky-2y\\ tx=y^2-k\end{matrix}\right.$

Nếu $y^2-k>0:$

Dễ thấy $t>0\Longrightarrow t\geqslant x\geqslant y$ (vì $x+y$ nhỏ nhất)

$\Longrightarrow t+x=4ky-2y\leqslant 2t \iff 2kxy-xy\leqslant tx$

$\iff 2kxy-xy\leqslant y^2-k \iff k\leqslant \frac{y^2+xy}{2xy+1}\leqslant 1$

$\Longrightarrow k=1\iff n=1$ (vô lí)

Nếu $y^2-k<0\Longrightarrow t<0$ 

$\Longrightarrow t^2+t(2y-4ky)+y^2-k=(t+y)^2-k(1+4ty)>0$ (vô lí vì $t$ là nghiệm của $(2)$)

Vậy $y^2-k=0\iff k=y^2$ là số chính phương (đpcm)




#646957 $m^2+7p^2=2^n$

Posted by Minhnguyenthe333 on 28-07-2016 - 22:01 in Số học

TH1: $n$ chẵn thì đặt $n=2k $
Sau đó đưa về $2^{2n} - m^2 = 7p^2 $ rồi trâu bò xét trường hợp
TH2: $n$ lẻ thì đặt $n=2k+1$
Xét $p \geq 3$
$m^2 + 7p^2 = 2.4^{k} $
Dễ thấy $m,p$ cùng tính chẵn lẽ, khi đó $m,p$ cùng lẻ 
Đặt $m=2a+1 , p =2b+1 $
Khi đó, ta được $4a^2+4a+1 + 4b^2+4b+1 = 2.4^k $
                          $4a(a+1) +4b(b+1) +2 = 2.4^k $
Nếu $k \geq 1$ thì $VP \vdots 4 $, còn VT thì không 
Do đó suy ra vô lí
Do đó $k=0 => 2a(a+1) + 2b(b+1)=0 $ cũng vô lí
Do đó $p=2 $
Khi $p=2$, ta được $m^2+28=2^n, n \geq 5 $
Tới đây xét tính
Phần cuối làm hơi dài nên không tiện ghi

Cái chính là bạn giải kĩ đoạn xét từng trường hợp, đó mới là phần khó của bài toán...
Bài này hình như là có $7$ bộ nghiệm



#646916 $m^2+7p^2=2^n$

Posted by Minhnguyenthe333 on 28-07-2016 - 18:28 in Số học

Tìm bộ ba số nguyên dương $(m,n,p)$ với $p$ là số nguyên tố sao cho:  $m^2+7p^2=2^n$

                                                          




#646779 $MQ,NR,PS$ đồng quy

Posted by Minhnguyenthe333 on 27-07-2016 - 19:50 in Hình học

Cho hai tam giác $ABC,DEF$ cùng nội tiếp một đường tròn sao cho phần giao của chúng tạo thành lục giác $MNPQRS$.Chứng minh rằng $MQ,NR,PS$ đồng quy




#646758 Tìm tất cả các số nguyên tố p và các số nguyên dương x,y thỏa mãn: $...

Posted by Minhnguyenthe333 on 27-07-2016 - 17:03 in Số học

Tìm tất cả các số nguyên tố p và các số nguyên dương x,y thỏa mãn: $\frac{x^3y}{x+y}=p$

Chợt nhận ra bài làm mình sai đoạn $x+y \mid x^2y^2<=>x+y\mid xy$

Đặt $d=\gcd(x,y)$, khi đó tồn tại 2 số $m,n$ thỏa $\gcd(m,n)=1$ sao cho $x=dm$ và $y=dn$

$=>p=\frac{d^3m^3n}{m+n}=p<=>d^3m^3n=pm+pn$ $(*)$

Dễ thấy $n \mid pm$ và $m \mid pn$

$TH1: n\mid pm$

Ta có $\gcd(m.n)=1$ nên $n\mid p$ hoặc $p\mid n$

Nếu thì $n\mid p$ thì $n=1$ vì $n=p$ kéo theo $n\mid m$ (vô lí)

Thay vào $(*)$ suy ra $d^3m^3=pm+p$ kéo theo $m\mid p$

$m=1=>d^3=2p$ (loại)

$m=p=>d^3=\frac{p+1}{p^2}$ (vô nghiệm)

$TH2: m\mid pn$

Tương tự như trên ta suy ra được $m=1$ 

$(*)<=>d^3n=p+pn$ kéo theo $n\mid p$

$n=1=>d^3=2p$ (loại)

$n=p=>d^3-1=(d-1)(d^2+d+1)=p$ kéo theo $d=2$ và $p=7$

Khi đó $(x,y,p)=(2,14,7)$




#646628 $2-\frac{1}{n}< \alpha < 2$

Posted by Minhnguyenthe333 on 26-07-2016 - 21:00 in Số học

Giả sử $\alpha$ là nghiệm dương của phương trình: $x^n=x^{n-1}+x^{n-2}+...+x+1$ $(n\geqslant 2, n\in \mathbb{Z^+})$

Chứng minh rằng: $2-\frac{1}{n}< \alpha < 2$




#646579 $a^2-b\mid ab$

Posted by Minhnguyenthe333 on 26-07-2016 - 17:11 in Số học

Đặt $z=\frac{ab}{a^2-b}$
$a^2-b\mid ab<=>ab\geqslant a^2-b<=>b\geqslant \frac{a^2}{a+1}$
Kéo theo $b\geqslant \left \lceil \frac{a^2}{a+1} \right \rceil=a$
TH1:$ a \mid b$
Ta đặt $b=ka$ suy ra $z=\frac{ak}{a-k}<=>\frac{1}{z}+\frac{1}{a}=\frac{1}{k}$$<=>a\mid zk$ $<=>\left\{\begin{matrix} a\mid z\\ \frac{a}{k} \mid z \end{matrix}\right.$
Bổ đề:
Spoiler

Chứng minh:
Spoiler

Áp dụng bổ đề trên:
Nếu $a\mid z$ thì ta đặt $d=\gcd(a,k)$
$d=1$ kéo theo $z=a(a-1)$ hay $(a,b)=(l,l^2-l)$ với mọi $l\in \mathbb{Z^+}$
$d\neq 1$ kéo theo $(a,b)=(l,\frac{l^2(v-1)}{v})$ với $l,v \in \mathbb{Z^+}$

Nếu $\frac{a}{k} \mid z$ tức $k\mid a$ thì tồn tại số $x$ sao cho $kx=a<=>b=k^2x$
$=>z=\frac{kx}{x-1}<=>x-1 \mid k$ nên $(a,b)=(\frac{x^2}{h}+x,\frac{x^3}{h}+x^2)$
TH2: $a \nmid b$ và $(a,b)=1$
Ta có $a^2-b \mid ab<=> a^2-b \mid a^3$
Đặt $t=\frac{a^3}{a^2-b}<=>a^3=ta^2-tb$ kéo theo $a^2\mid kb$
Do $(a,b)=1$ nên $a^2\mid t$ hoặc $t\mid a^2$
Nếu $a^2\mid t:$
$=>a^2-b \mid a$ kéo theo $a^2-b=\pm 1$ hay $(a,b)=(m,m^2+1);(m,m^2-1)$
Nếu $t\mid a^2:$
$=>\frac{a^2}{t}=a-\frac{b}{a}$ kéo theo $a \mid b$ (vô lí)

Vậy$...$



#646288 $a^2-b\mid ab$

Posted by Minhnguyenthe333 on 24-07-2016 - 20:19 in Số học

(AoPS) Tìm cặp $(a,b)$ nguyên dương sao cho $a^2-b\mid ab$

P/S: bài này hay :)




#646278 Chứng minh rằng $a=b.$

Posted by Minhnguyenthe333 on 24-07-2016 - 18:24 in Số học

Cho các số nguyên dương $a,b$ thỏa mãn $\frac{ab(5a^{2}+5b^{2}-2)}{5ab-1}\in\mathbb{Z}$. Chứng minh rằng $a=b$.

Xem tại đây: https://julielltv.wo...uoc-nhay-viete/
Cách của mình:
Do $(ab,5ab-1)=1$ nên $5ab-1\mid 5a^2+5b^2-2$
Đặt $k=\frac{5a^2+5b^2-2}{5ab-1}$ $<=>5a^2-5bka+5b^2+k-2=0$
Cố định tập nghiệm, giả sử $a\geqslant b$ và $a+b$ min
Theo Vieta, tồn tại thêm nghiệm nguyên $t$ sao cho: $\left\{\begin{matrix}t+a=bk \\ 5ta=5b^2+k-2 \end{matrix}\right.$
Suy ra $t>0$ kéo theo $5bk\leqslant 10t<=>5abk\leqslant 10ta=10b^2+2k-4$
Chú ý rằng $a\geqslant b$ nên $10b^2+2k-4\geqslant 5b^2k$
$<=>(5b^2-2)k\leqslant 10b^2-4=2(5b^2-2)$ kéo theo $k\leqslant 2$
Mặt khác ta có $k\geqslant 2<=> 5(a-b)^2\geqslant 0$ (luôn đúng)
Do đó $k=2$ hay $5(a-b)^2=0$ kéo theo $a=b$ (đpcm)



#646235 $x^2+y^2+x+y-kxy=0$

Posted by Minhnguyenthe333 on 24-07-2016 - 15:27 in Số học

Tìm tất cả k nguyên dương để phương trình sau có nghiệm nguyên dương

$x^2+y^2+x+y-kxy=0$

$PT<=>x^2+x(1-ky)+y^2+y=0$

Cố định tập nghiệm, giả sử $x\geqslant y$ và $x+y$ min

Theo định lý Viete, tồn tại thêm nghiệm nguyên $t$ sao cho: $\left\{\begin{matrix}t+x=ky-1\\ tx=y^2+y\end{matrix}\right.$

Dễ thấy $t>0$ nên $t\geqslant x\geqslant y$

Khi đó $ky-1=t+x\leqslant 2t<=>x(ky-1)\leqslant 2tx$ hay $x(ky-1)\leqslant 2(y^2+y)$

Chú ý rằng $x\geqslant y$ $=>$ $2(y^2+y)\geqslant y(ky-1)$

$<=>3y\geqslant (k-2)y^2<=>k-2\leqslant \frac{3}{y}\leqslant 3$ kéo theo $k\leqslant 5$

Xét từng trường hợp suy ra $.....$




#646211 $R_{1}=20\Omega ; R_{3}=R_{4}=10\Omega; R_{2}=15\Omega; U...

Posted by Minhnguyenthe333 on 24-07-2016 - 10:20 in Các môn tự nhiên (Vật lý, Hóa học, Sinh học, Công nghệ)

Cho mạch điện như hình vẽ:

 attachicon.gif untitled.JPG

Trong đó $R_{1}=20\Omega ; R_{3}=R_{4}=10\Omega; R_{2}=15\Omega; U_{AB}=20V.$

Tính dòng điện qua Ampe kế (Cho $R_{A}=0$)

Do $M\equiv B$ nên ta có sđmđ: $[R_2$ $nt$ $(R_3//R_4)]$ $//R_1$

Ta có: $I_1=\frac{U}{R_1}$

Tính được $R_{tđ}\rightarrow$ tính được $I_2=\frac{U}{R_{tđ}}-I_1$

Khi đó $U_2=I_2R_2\rightarrow U_3=U-U_2$

Tính được $I_3$ và ta có số chỉ ampe kế là $I_A=I_1+I_3$ 




#646181 Chứng minh $a$ là số chính phương

Posted by Minhnguyenthe333 on 23-07-2016 - 22:14 in Số học

Cho $a,b,c,d$ nguyên dương thỏa $a<b\leq c<d$ và $ad=bc$ , ngoài ra $\sqrt{d}-\sqrt{a}<1$ . Chứng minh rằng $a$ là số chính phương .

Em nghĩ đề phải là $\sqrt{d}-\sqrt{a}\geqslant 1$

Giả thiết$<=>\frac{a}{b}=\frac{c}{d}=\frac{m}{n}$ với $(m,n)=1$ $(n>m)$

Khi đó tồn tại 2 số nguyên dương $u,v$ thỏa $u>v$ sao cho $a=um$ và $d=(u+v)n$, $b=un$ và $c=(u+v)m$

Nếu $\sqrt{d}-\sqrt{a}<1$ thì $d<a+2\sqrt{a}+1$

$<=>(u+v)n<um+1+2\sqrt{um}<=>u(n-m)+nv-1<2\sqrt{um}$

Dễ thấy $2\sqrt{um}\leqslant u+m<u(n-m)+nv-1$ (mâu thuẫn)

Do đó $\sqrt{d}-\sqrt{a}\geqslant 1....$




#646152 $\frac{x+y+z}{3\sqrt{3}}\ge...

Posted by Minhnguyenthe333 on 23-07-2016 - 18:41 in Bất đẳng thức và cực trị

Cho $x,y,z>0$, chứng minh rằng:

       $\frac{x+y+z}{3\sqrt{3}}\geqslant \frac{xy+yz+zx}{\sqrt{x^2+xy+y^2}+\sqrt{y^2+yz+z^2}+\sqrt{z^2+zx+x^2}}$




#646045 Tổng 3 số bất kì lớn 2 số còn lại. Tìm GTNN của tổng 5 số đó.

Posted by Minhnguyenthe333 on 22-07-2016 - 20:49 in Số học

Với 5 số nguyên dương phân biệt có tính chất tổng 3 số bất kì lớn hơn tổng 2 số còn lại, tìm giá trị nhỏ nhất của tổng 5 số này.

Giả sử $a>b>c>d>e$ suy ra $b-d\geqslant 2$ và $a-c\geqslant 2$

Dễ thấy $a+d+e>b+c<=>e>(b-d)+(a-c)\geqslant 4$ kéo theo $e\geqslant 5$

Xét $e=5:$

Ta có: $\left\{\begin{matrix}d+e+a>b+c\\ d+e+b>a+c\end{matrix}\right.<=>d+e>c<=>1\leqslant c-d\leqslant 4$

Xét $d=6$ thì $7\leqslant c\leqslant 10$,chọn $c=7$ kéo theo $b\geqslant 8$

Chọn $b=8$ suy ra $a=9$ và ta được $(a,b,c,d,e)=(5,6,7,8,9)$ thỏa mãn

Do đó tồn tại bộ 5 số liên tiếp nên $S=a+b+c+d+e\geqslant 5+6+7+8+9=35$

DBXR khi $(a,b,c,d,e)=(5,6,7,8,9)$ và các hoán vị




#646038 Chứng minh: $\sum \dfrac{a}{\sqrt{1+a...

Posted by Minhnguyenthe333 on 22-07-2016 - 19:41 in Bất đẳng thức và cực trị

Cho $a,b,c$ là các số dương thỏa mãn $abc=1$. Chứng minh rằng: $$\dfrac{a}{\sqrt{1+a}}+\dfrac{b}{\sqrt{1+b}}+\dfrac{c }{\sqrt{1+c}}\geq \dfrac{3\sqrt{2}}{2}.$$

Biến đổi $(a,b,c)\rightarrow (\frac{x}{y},\frac{y}{z},\frac{z}{x})$

BĐT$<=>\sum \frac{x}{\sqrt{y(x+y)}}\geqslant \frac{3\sqrt{2}}{2}<=>\sum \frac{x}{\sqrt{2y(x+y)}}\geqslant \frac{3}{2}$

Áp dụng bđt Cauchy cho mẫu: $\sum \frac{x}{\sqrt{2y(x+y)}}\geqslant \sum \frac{2x}{x+3y}$

Theo C-S: $\sum \frac{2x}{x+3y}\geqslant \frac{2(x+y+z)^2}{(x+y+z)^2+xy+yz+zx}\geqslant  \frac{2(x+y+z)^2}{(x+y+z)^2+\frac{(x+y+z)^2}{3}}=\frac{3}{2}$ (đpcm)
DBXR khi $a=b=c=1$




#645912 $x^2+y^2+z^2=3^{2^n}$

Posted by Minhnguyenthe333 on 21-07-2016 - 19:50 in Số học

Định lý Legendre: Phương trình $n=u^2+v^2+w^2$ có nghiệm nguyên khi và chỉ khi $n\neq 4^s(8l+7)$

Áp dụng vào bài toán:

Đặt $d=GCD(a,b,c)$ suy ra $\exists$ $x,y,z$ sao cho $a=dx,b=dy,c=dz$ với $GCD(x,y,z)=1$

Dễ thấy $3^{2^n}d^2=4^s(8l+7)<=>8l+7$ là số chính phương kéo theo $8l+7\equiv 1$ $(mod$ $8)$ (vô lí)

Suy ra $3^{2^n}d^2\neq 4^s(8l+7)$ nên theo định lý ta có: $a^2+b^2+c^2=3^{2^n}d^2$

$<=>d^2(x^2+y^2+z^2)=3^{2^n}d^2<=>x^2+y^2+z^2=3^{2^n}$ 

Do $GCD(x,y,z)=1$ nên ta có đpcm




#645824 Chứng minh rằng $\Delta ABC$ và $\Delta DEF$...

Posted by Minhnguyenthe333 on 21-07-2016 - 11:19 in Hình học

Cho $\Delta ABC$. Tren các cạnh $AB, BC, CA$ lần lượt lấy các điểm $D, E, F$ sao cho $\frac{AD}{AB}=\frac{BE}{BC}=\frac{CF}{CA}=k(0<\frac{1}{2})$. Chứng minh rằng $\Delta ABC$ và $\Delta DEF$ có chung trọng tâm.
(Khuyến khích các bạn giải bằng nhiều cách và nêu hướng giải thì càng tốt)

Bổ đề: $\Delta ABC$ và $\Delta A'B'C'$ có cùng trọng tâm khi và chỉ khi $\sum \overrightarrow{AA'}=0$
Spoiler


Từ giả thiết ta có $AD=kAB<=> \overrightarrow{AD}=k \overrightarrow{AB}$
Áp dụng bổ đề, ta cần chứng minh $\overrightarrow{AD}+\overrightarrow{BE}+\overrightarrow{CF}=0$
$<=>k(\overrightarrow{AB}+\overrightarrow{BC}+\overrightarrow{CA})=0$
$<=>k(\overrightarrow{AC}+\overrightarrow{CA})=0<=>\overrightarrow{AC}+\overrightarrow{CA}=0$
Điều này luôn đúng nên ta có đpcm



#645691 Chứng minh rằng $\sqrt{1+xy}$ là một số hữu tỉ.

Posted by Minhnguyenthe333 on 20-07-2016 - 18:47 in Đại số

Cho $x$, $y$ là các số hữu tỉ thoả mãn đẳng thức $x^2+y^2+\left ( \frac{xy+1}{x+y} \right )^2 = 2$. Chứng minh rằng $\sqrt{1+xy}$ là một số hữu tỉ.

Ta có: $x^2+y^2+\left ( \frac{xy+1}{x+y} \right )^2= 2$
$<=>(x+y)^2+\left ( \frac{xy+1}{x+y} \right )^2= 2(xy+1)$
$<=>(x+y)^2-2(xy+1)+\left ( \frac{xy+1}{x+y} \right )^2= 0$
$<=>(x+y-\frac{xy+1}{x+y})^2= 0$
$<=>(x+y)^2=xy+1<=>\sqrt{1+xy}=x+y$ là số hữu tỉ
Do đó ta có đpcm



#645663 $y^{2}=x^{3}+16.$

Posted by Minhnguyenthe333 on 20-07-2016 - 15:18 in Số học

Đoạn này mình nghĩ là thêm TH $y_3=-1$. Nhưng có vẻ không ảnh hưởng gì đến bài giải :)
 



Còn đoạn này thì $TH$ $x$ có dạng $4K+1$ suy được $y^{2}\equiv 1(mod4)$ thì đâu sai?

Mình bổ sung thêm TH $x=4k+1$
Khi đó $y$ lẻ và $PT<=>x^3=(y-4)(y+4)$
Đặt $d=GCD(y-4,y+4)$
Dễ thấy $d$ lẻ và $d\mid 8$ kéo theo $d=1$
Khi đó $y+4=m^3$ và $y-4=n^3$
$<=>m^3-n^3=8<=>(m-n)(m^2+mn+n^2)=1.8=2.4$
$=>(m,n)=(0,-2);(2,0)$ (loại vì $m,n$ lẻ)